Solve for n.

11(n – 1) + 35 = 3n

Answers

Answer 1

Answer:

n = -3

Step-by-step explanation:

11(n – 1) + 35 = 3n

Distribute

11n -11 +35 = 3n

Combine like terms

11n +24 = 3n

Subtract 11n from each side

11n+24 -11n = 3n -11n

24 = -8n

Divide by -8

24/-8 = -8n=-8

-3 =n

Answer 2
Answer: -3=n

Step 1: Rewrite equation

11(n-1)+35=3n

Step 2: Distribute

To distribute, you need to multiply the numbers on the outside of the parentheses with the numbers on the inside. To do so, you need to multiple 11•n and 11•-1.

11(n-1)+35=3n
11n-11+35=3n

Step 3: Combine like terms

“Like terms” are numbers that share the same variable, or lack of. To start, you must only combine like terms on the same side of the equation. In this case, the like terms are -11 and 35. Add these to combine like terms.

11n-11+35=3n
11n+24=3n

Step 4: Put variables on one side

It is important to keep variables on one side of the equation, and the constants on the other. To do so, we need to subtract 11n on both sides.

11n+24=3n
24=-8n

Step 5: Get n alone

In this final step, we need to get n alone to solve for the variable. To do so, we need to get rid of the -8 next to it. We can do this by diving 24 by -8 and diving -8n by -8.

24=-8n
-3=n

This is your final answer: 3=n. Hope this helps! Comment below for more questions.

Related Questions

Simplify 6(x + 3). i suck at math pls help

Answers

Answer:

6x +18

Step-by-step explanation:

6(x + 3)

Distribute

6*x + 6*3

6x +18

Answer:

6x + 18

Step-by-step explanation:

Given

6(x + 3) ← multiply each term in the parenthesis by 6

= 6x + 18

Harriet and Maya share £300 in the ratio of 7:5. Wirk out how much money harriet gets

Answers

Answer:

$175

Step-by-step explanation:

you need to specify if harriet got the 7 portion or the 5 portion. I'll answer as if she got the 7 portion.

They split it into 12 portions because the ratio is 7:5 and 7+5=12. So do $300/12=25

Assuming Maya got the 5 portion, do $25 x 5 = $125, so Maya got $125.

Assuming Harriet got the 7 portion, do $25 x 7 = $175, so Harriet got $175.

Will mark brainliest!!!

Answers

Answer:

option C , D , E

Step-by-step explanation:

[tex]\sqrt{\frac{1}{16}} = \frac{1}{4} => not \ irrational\\\\\sqrt{\frac{1}{4}} = \frac{1}{2} => not \ irrational\\\\\sqrt{\frac{1}{2}} = > \ irrational\\\\\sqrt{\frac{1}{10}} =>\ irrational\\\\\sqrt{\frac{1}{8}} =>\ irrational\\\\[/tex]

Answer:

1/2, 1/10, 1/8

I need help with 27 and 29 plz

Answers

27) C
29) B
Remember when you are after a decimal it’s . Tenths, hundredths, thousandths

Answer:

27.C

29.B

Step-by-step explanation:

The area of a rectangle is expressed as (15x + 20) square feet. If the width of the rectangle is 5 feet, what is an expression to represent the length of the rectangle.
(No picture)

Answers

Answer: Length = (15x + 20) / 5

Step-by-step explanation:

The area of a rectangle is calculated as length × width. Since the area of the rectangle is expressed as (15x + 20) square feet while the width of the rectangle is 5 feet, then the expression to represent the length of the rectangle will be:

Area = length × width

(15x + 20) = length × 5

Length = (15x + 20) / 5

Therefore, the expression is Length = (15x + 20) / 5.

Solving further, the length will be:

= (15x + 20) / 5

= 3x + 4

Please help me solve this

Answers

Answer:

5890 (none of the above)

Step-by-step explanation:

Answer:

1472.6 in³

Step-by-step explanation:

The volume (V) of the oblique cone is calculated as

V = [tex]\frac{1}{3}[/tex] πr²h ( r is the radius and h the perpendicular height )

Here r = 15 ÷ 2 = 7.5 and h = 25 , then

V = [tex]\frac{1}{3}[/tex] × π × 7.5² × 25

   = [tex]\frac{1}{3}[/tex] × π × 56.25 × 25

   = [tex]\frac{1}{3}[/tex] × π × 1406.25

   = [tex]\frac{1406.25\pi }{3}[/tex] ≈ 1472.6 in³ ( to the nearest tenth )

help asap ---------------------------

Answers

Answer:

"D"

the start point is below zero and is doing deeper

Step-by-step explanation:

If θ is an angle in standard position and its terminal side passes through the point (-4,-9), find the exact value of \tan\thetatanθ in simplest radical form.

Answers

Given:

θ is an angle in standard position and its terminal side passes through the point (-4,-9).

To find:

The exact value of tanθ in simplest radical form.

Solution:

If θ is an angle in standard position and its terminal side passes through the point (x,y), then the exact value of tanθ is:

[tex]\tan \theta=\dfrac{y}{x}[/tex]

It is given that, θ is an angle in standard position and its terminal side passes through the point (-4,-9). So, t he exact value of tanθ is:

[tex]\tan \theta=\dfrac{-9}{-4}[/tex]

[tex]\tan \theta=\dfrac{9}{4}[/tex]

Therefore, the required value is [tex]\tan \theta=\dfrac{9}{4}[/tex].

Jen's class 10 girls and 15 boys. The ratio of girls to boys in Ed's class is the same as the ratio of girls to boys in Jen's class. There are 24 boys in Ed's class. How many girls are in Ed's class?

Answers

Answer:

16

Step-by-step explanation:

Jen's class: 10 girls, 15 boys

ratio of girls to boys = 10:15 = 2:3 = 2x:3x

Ed's class

number of boys = 24

ratio of girls to boys = 2x:3x

3x = 24

x = 8

2x = 2(8) = 16

Answer: 16

Answer:

16

Step-by-step explanation:

Create a linear system to model this situation. Then use substitution to solve the linear system to solve
the problem:
Bobbie has been saving dimes and quarters to buy a new toy. She has a total of 45 dimes and quarters,
with a value of $7.05. How many of each type of coin does Bobbie have?

Answers

Answer:

dimes --- x

quarters ---- 28-x

10x + 25(28-x) = 430

Step-by-step explanation:

not sure po

help..................​

Answers

Answer:

1. 6084

2. 9261

3. -1

4. 995/364

5. -1/625

Step-by-step explanation:

THE LENGTH OF THE CAR IS 435 CM. THE LENGTH F THE GARAGE IS 6 M. HOW MANY CENTIMETERS LONGER IS THE GARAGE THAN THE CAR?

Answers

Answer:

235cm longer

Step-by-step explanation:

convert 6m into centimeters which gets you 600cm so you do this:

600 - 435= 235cm

therefore the garage is 235cm longer than the car.

Answer:

600-435 = 165 cm the garage 165 cm is longer than the car

Find f(3) given f(x) = -3x^3 + 2x^2 + 24
A. 123
B. -39
C. 69
D. 99

Answers

The correct answer is d

7/9 × 27/5 whats the answer?​

Answers

Answer:

21/5 when simplified

Answer:

4  1/5

Step-by-step explanation:   6  20  5  58

7/9 × 27/5  =  (7×27) / (9 × 5)

                 =   189 / 45           divide numerator and denominator by 3

                 =    63 / 15            simplify

                 =     4 3/15

                 =     4  1/5

Which is a valid conclusion that can be drawn from these statements?

If a quadrilateral is a rhombus, then it is a parallelogram.
If a quadrilateral is a parallelogram, then its opposite angles are congruent.

A. Opposite angles of a rhombus are congruent.

B.Opposite angles of a quadrilateral are congruent.

C. Every parallelogram is a rhombus.

D. Every quadrilateral is a rhombus.

Answers

Answer:

A. Opposite angles of a rhombus are congruent.

Step-by-step explanation:

The height of a certain plant can be modeled by the formula h = 4 t + 5 where h is the height in millimeters and t is the time in days.

Answers

Answer:

The dependent variable is h and the independent variable is t

Step-by-step explanation:

Given

[tex]h(t) = 4t + 5[/tex]

Required

The dependent and the independent variable

The [tex]dependent[/tex] [tex]variable[/tex]is the [tex]variable[/tex] whose value [tex]depends[/tex] on another.

In the above equation, the height (h) depends on time (t) for its value

Hence,

The [tex]depe ndent[/tex] [tex]variable[/tex] is h and the [tex]indep endent[/tex] [tex]variable[/tex] is t

(6.6 × 10⁰) + (7.8 × 10²)​

Answers

Answer:

786.6

Step-by-step explanation:

Anything to the ^0 is 1 so its 1 x 6.6 + 100 x 7.8

6.6 + 780

786.6

Answer:

786.6

Step-by-step explanation:

Problem:

(6.6 × 10⁰) + (7.8 × 10²)​

Step 1 - Parenthesis:

(6.6 × 10⁰) + (7.8 × 10²)​

Step 2 - Exponents(Left to Right):

(6.6 × 10^0) + (7.8 × 10^2)​ = (6.6 × 1) + (7.8 × 100)

Step 3 - Multiply(Left to Right):

(6.6 × 1) + (7.8 × 100) = 6.6 + 780

Step 4 - Add(Left to Right)

6.6 + 780 = 786.6

Step 5 - Answer:

786.6

Remember:

Parenthesis

Exponents(Left to Right)

Multiplication(Left to Right)

Division(Left to Right)

Addition(Left to Right)

Subtraction(Left to Right)

20 men take 10 days to complete a piece of work. find the time taken by 8 men to complete the same piece of work

Answers

For the 8 men to finish the work it would take 16.76 days to finish the piece of work

The masses to the nearest kilogram of nine men were:

75, 68, 78, 82, 85, 90, 88, 92, 76.

Calculate the mean mass​

Answers

Answer:

82 kg

Step-by-step explanation:

To find the mean of a set of values, we must add them up and divide by the number of values.

Step 1, adding the values:

[tex]75+68+78+82+85+90+88+92+76=\\734[/tex]

Step 2, dividing by the # of terms:

There are 9 terms in total.

[tex]\frac{734}{9} =\\81.556[/tex]

To the nearest kilogram, we can round 81.556 into 82.

The mean mass of the 9 men was [tex]\fbox{82}[/tex] kg.

I hope this helps! Let me know if you have any questions :)

82kg is the answer I did research and I’m 90% sure that the answer

please help…………..i have more to ask

Answers

Answer:

YES

Step-by-step explanation:

Find each length using the distance formula, [tex] d = \sqrt{(x_2 - x_1)^2 + (y_2 - y_1)^2} [/tex]

✔️Distance between A(-4, 2) and B(1, 4):

[tex] AB = \sqrt{(x_2 - x_1)^2 + (y_2 - y_1)^2} [/tex]

Let,

[tex] A(-4, 2) = (x_1, y_1) [/tex]

[tex] B(1, 4) = (x_2, y_2) [/tex]

[tex] AB = \sqrt{(1 - (-4))^2 + (4 - 2)^2} [/tex]

[tex] AB = \sqrt{(5)^2 + (2)^2} [/tex]

[tex] AB = \sqrt{25 + 4} [/tex]

[tex] AB = \sqrt{29} [/tex]

✔️Distance between C(2, -1) and D(4, 4):

[tex] CD = \sqrt{(x_2 - x_1)^2 + (y_2 - y_1)^2} [/tex]

Let,

[tex] C(2, -1) = (x_1, y_1) [/tex]

[tex] D(4, 4) = (x_2, y_2) [/tex]

[tex] CD = \sqrt{(4 - 2)^2 + (4 - (-1))^2} [/tex]

[tex] CD = \sqrt{(2)^2 + (5)^2} [/tex]

[tex] CD = \sqrt{4 + 25} [/tex]

[tex] CD = \sqrt{29} [/tex]

AB = CD = √29

Therefore, they both have the same length.

Y=x+5 y = x + 5 Find four points contained in the inverse. Express your values as an integer or simplified fraction.

Answers

Answer: (0,-5), (5, 0), (10, 5) and (15,10).

Step-by-step explanation:

Given equation: [tex]y = x + 5[/tex]

To find its inverse we will find the value of x .

Subtract 5 from both sides

[tex]x=y-5[/tex]

Switch y to x, we get

y= x-5, which is the inverse function.

At x=0,

y=-5

At x= 5,

y=0

At x= 10 ,

y=5

At x= 15,

y= 10

Hence, the four points contained in the inverse: (0,-5), (5, 0), (10, 5) and (15,10).

Plotting the graph after finding the inverse function the four points contained in the inverse fucntion are:  (0, -5), (5, 0), (5.1, 0.1), and (5.2, 0.2).

The given equation is,

y = x+5

To find its inverse fucntion,

Replace x with y we get.

x = y + 5

Rearranging it,

y = x - 5

So this is the inverse fucntion.

Since this is of the form of line y = mx + c

Therefore,

To find the points containing contained by inverse fucntion,

Graphing: y = x - 5

After graphing we can see that,

The four points (0, -5), (5, 0), (5.1, 0.1), and (5.2, 0.2) are lies on the line.

Hence the required four points are:

(0, -5), (5, 0), (5.1, 0.1), and (5.2, 0.2)

#SPJ6

what are the steps to this?

Answers

Answer:

Start at the positive

x

-axis, then rotate left by the desired angle.

Explanation:

Standard position means the first arm of the angle is the positive

x

-axis, and the other arm is placed by rotating counter-clockwise from there, by the amount of the angle.

As a basic example, the symbol

is about a 45° angle in standard position.

To get a feel for where the second arm (called the "terminal arm") will go, remind yourself that the axes themselves meet each other at 90°.

If our angle was 90°, the terminal arm would be on the positive

y

-axis.

If our angle was 180°, it would be on the negative

x

-axis.

Wait! 180° is more than 150°, so our angle is somewhere in quadrant 2. In fact, 150° is 2/3 of the way between 90° and 180°, so our terminal arm will be 2/3 of the way into quadrant 2.

graph{(y+tan(pi/6)x)(y^2-.00001x)=0 [-10, 10, -5, 5]}

(ignore the part of the line in quadrant 4)

Step-by-step explanation:

Are the following triangles similar, Explain your answer​

Answers

Yes the have the same 78° angle . The smaller one should be able to fit inside the larger one.

Each of 3 water sprinkler systems covers a semicircle with radius 2m on the

side of a house shown.

What amount of area remains dry? Show your work to explain how you go

your answer, to the nearest square meter. (Dry area is in the shaded

region.)

Answers

Answer:

5 meters

Step-by-step explanation:

We can obtain the entire area, both shaded and non-shaded by finding the area of the rectangle ;

Area of rectangle = Length * width

Length = 12 ; width = 2

Area of rectangle = 12 * 2 = 24 m²

Area of semicircle = πr² / 2

The radius, r of each semicircle = 2

Area of each semicircle = π * 2² / 2 = 6.283 m²

Area of the 3 semicircles = 3 * 6.283 = 18.849 m²

Area of dry region:

(24 - 18.849) m²

= 5.151 m²

= 5 m

The product of two integers is (-112).
If one of them is (-8), find the other.​

Answers

Answer:

14

Step-by-step explanation:

To find the other number , divide -112 by -8 and you will get 14.

https://brainly.in/question/1284951 - see this link have a answer ;)

What is 2.3(x – 2) = 15

Answers

[tex]\huge\textsf{Hey there!}[/tex]

[tex]\mathsf{2.3(x - 2) = 15}[/tex]

[tex]\mathsf{2.3(x) + 2.3(-2) = 15}[/tex]

[tex]\mathsf{2.3x - 4.6 = 15}[/tex]

[tex]\large\textsf{ADD 4.6 to BOTH SIDES}[/tex]

[tex]\mathsf{2.3x - 4.6 + 4.6 = 15 + 4.6}[/tex]

[tex]\large\textsf{CANCEL out: -4.6 + 4.6 because it gives the number 0}[/tex]

[tex]\large\textsf{KEEP: 15 + 4.6 because that helps solve find the x-value}[/tex]

[tex]\mathsf{15 + 4.6 = \boxed{\bf 19.6}}[/tex]

[tex]\large\textsf{NEW EQUATION: 2.3x = 19.6}[/tex]

[tex]\large\textsf{DIVIDE 2.3 to BOTH SIDES}[/tex]

[tex]\mathsf{\dfrac{2.3x}{2.3}=\dfrac{19.6}{2.3}}[/tex]

[tex]\large\textsf{CANCEL out: }\mathsf{\dfrac{2.3}{2.3}}\large\textsf{ because that gives you 1}[/tex]

[tex]\large\textsf{KEEP: }\mathsf{\dfrac{19.6}{2.3}}\large\textsf{ because that gives you the value of x}[/tex]

[tex]\mathsf{\dfrac{19.6}{2.3}= \boxed{\bf x}}[/tex]

[tex]\mathsf{\boxed{\bf x}=\dfrac{19.6}{2.3}}[/tex]

[tex]\boxed{\bf {x = 8.521739}}[/tex]

[tex]\boxed{\boxed{\large\textsf{Answer: \huge \bf x = 8.521739}}}\huge\checkmark[/tex]

[tex]\large\textsf{Good luck on your assignment and enjoy your day!}[/tex]

~[tex]\frak{Amphitrite1040:) }[/tex]

jsfghsjgehgeujvnegeuge

Answers

Answer:

Option 2: 67.014

Step-by-step explanation:

PLEASEEE HELPPPPPPP!!!!!!! question is above

Answers

AB=5

Step-by-step explanation:

x^2 - 10=3x

x^2 -3x -10=0

x^2 +2x-5x-10=0

x(x+2) -5(x+2) =0

(x-5) (x+2) =0

x-5=0. x+2=0

x=5. x=-2

Question 8
A scuba diver was thirty feet below the surface of the water and ascended
thirteen and a half feet. The diver then descended an additional fifteen feet
from the surface of the water. What is the location of the diver in the water?
a.-48 feet
b.-31.5 feet
c. - 27.5 feet
X d. -28 feet

Answers

It’s b

Explanation

The scuba diver is 30 feet below, and ascend means up, so you subtract 13.5, and descend means down down so you add 15. That’s how you get the answer.

Answer:

b

Step-by-step explanation:

The scuba diver is 30 feet below, and ascend means up, so you subtract 13.5, and descend means down so you add 15. That’s how you get the answer.

( 3x + 1 ) ( x - 4 ) - ( x + 2 ) ( x - 3 ) = ( 2x + 5 ) x
Giúp tớ tính với

Answers

[tex]\sf \bf {\boxed {\mathbb {x\:=\:0.1333}}}[/tex]

[tex]\large\mathfrak{{\pmb{\underline{\red{Step-by-step\:explanation}}{\red{:}}}}}[/tex]

[tex](3x + 1)(x - 4) - (x + 2)(x - 3) = (2x + 5)x\\[/tex]

[tex] ➺\: 3x \: (x - 4) + 1 \: (x - 4) - x \: (x - 3) - 2 \: (x - 3) = (2x + 5) \: x\\[/tex]

[tex] ➺\: 3 {x}^{2} - 12x + x - 4 - {x}^{2} + 3x - 2x + 6 = 2 {x}^{2} + 5x\\[/tex]

[tex] ➺\: 2{x}^{2} - 2 {x}^{2} - 10x - 5x + 2= 0\\[/tex]

[tex] ➺\: - 15x = - 2\\[/tex]

[tex]➺ \: x = \frac{ - 2}{ - 15} \\[/tex]

[tex]➺ \: x = \frac{2}{15}\\ [/tex]

[tex] ➺\: x = 0.1333[/tex]

[tex]\huge\bf\purple{*Mystique 35♡༉}[/tex]

Other Questions
Which number(s) below belong to the solution set of the inequality? Check allthat apply.X+ 30 < 60A. 30B. 45C. 1D. 50E 300F 29 Suppose that past records indicate that the probability that a new car will need a warranty repair in the first 90 days of use is 0.04. If a random sample of 400 new cars is selected. what is the probability that the proportion of new cars needing a warranty repair in the first 90 days will be: a. between 0.05 and 0.06? i.e. P(0.05 SpS 0.06) = (round your answer to 4 decimal places). b. above 0.07? i.e. P(p > 0.07) = (round your answer to 5 decimal places) c. below 0.03?ie. P(p < 0.03) = (round your answer to 4 decimal places) Plisss help me plisssss If p and q are the roots of 2x+ 6x = 12 + 4x, and p < q, find q p I might give brainliest, please help down below. Why might it be important to be able to use text-based evidence in a project? if you have to leave it during this time, and you heard about gold in California would you have traveled west to California ? why or why not? what does unintentional and intentional injury mean? Multiply (5xy-4)(5xy+4) You are riding your bike and notice the square sign above. You mentally draw astraight line from point A to C. Describe the angle relationship between /_DCA and/_BCA. Which measurements could create more than one triangle? Information related to Kerber Co. is presented below.1. On April 5, purchased merchandise on account from Monty Company for $36,000, terms 3/10, net/30, FOB shipping point. 2. On April 6, paid freight costs of $920 on merchandise purchased from Monty. 3. On April 7, purchased equipment on account for $30,500. 4. On April 8, returned damaged merchandise to Monty Company and was granted a $4,200 credit for returned merchandise. 5. On April 15, paid the amount due to Monty Company in full.Required:Prepare the journal entries to record these transactions on the books of Riverbed co. under a perpetual inventory system. discuss two reasons why parents may feel frustrated when teenagers are constantly on their cell phones? NEED HELP ASAP So for this problem I got 10.8 by multiplying 0.60 x 18. However it stated that my answer is incorrect. How do I go about this problem because I am not sure what else to do? Can someone help me on this please. Solve the following quadratic equation. *x^2+12x-45=0 Factor completely 3x2 + 9x 3. 3(x2 + 3) 3(x2 + 3x 1) 3x(x2 + 3x 1) Prime a girl 65 marks out of 80. What percent mark did she have? Please help me!! I will mark you as BRAINLIEST ANSWER! Which data set has high consistency and low variability? Why? What is the slope of the line?Pls help